LSAT and Law School Admissions Forum

Get expert LSAT preparation and law school admissions advice from PowerScore Test Preparation.

 martinbeslu
  • Posts: 49
  • Joined: Aug 09, 2017
|
#44491
I'm not sure that I understand how answer choice A weakens the argument. It seems like an answer choice like this would normally be quickly disregarded. If the consultants have performed poorly over 12 years, then how is one good year relevant? What if the answer choice said the past month, or the past day, or the past 5 minutes?
 Shannon Parker
PowerScore Staff
  • PowerScore Staff
  • Posts: 147
  • Joined: Jun 08, 2016
|
#44520
martinbeslu wrote:I'm not sure that I understand how answer choice A weakens the argument. It seems like an answer choice like this would normally be quickly disregarded. If the consultants have performed poorly over 12 years, then how is one good year relevant? What if the answer choice said the past month, or the past day, or the past 5 minutes?
The key is that the conclusion states that no one should "ever" follow any recommendations by these so called experts. Even if the answer choice said "five minutes" it would still weaken the argument, although not very significantly, because it would be offering a scenario in which the experts were correct.

Hope this helps.
Shannon
 martinbeslu
  • Posts: 49
  • Joined: Aug 09, 2017
|
#44592
So I guess I'm confusing some previous lessons with something that isn't really applicable here. I viewed this as a part vs whole argument. I thought that we couldn't use some example from part of the set to weaken the whole.

I see what you are saying in that the conclusion states, no one should EVER follow the recommendations. However, the analogy that comes to mind is this: premise - driving drunk is very dangerous and 100,000 people have died in car accidents as a result over the last 12 years. conclusion - you should NEVER drive drunk. answer choice A - the last three people that drove home drunk this year did not die in car accidents.

I'm so lost on this one.
 Malila Robinson
PowerScore Staff
  • PowerScore Staff
  • Posts: 296
  • Joined: Feb 01, 2018
|
#44642
Hi Martin,
Remember that with a Weaken question any info (even if it is outside info) that would weaken the argument is correct. In this case we are not looking to completely disprove the argument, we just want to show that the very strongly worded conclusion that no one should EVER listen to the recommendations of the experts is wrong. So even if you can show that one person at one point in time, should listen to the recommendation you have already proven the conclusion wrong.

The example you gave doesn't really work because we wouldn't just be talking about the deaths of the drunk drivers, they could also live, but cause the death of others.
Hope that helps,
Malila
 HowardQ
  • Posts: 32
  • Joined: Jun 25, 2018
|
#48948
Hi,

I thought C was an attractive answer, since it mentions telephone show versus the market, it is not relevant at all with the passage. since weakenX includes no effect or not relevant, shouldn't C be a viable answer? while D shows a possible flaw in the determination of effectiveness. Shouldn't C be a better answer?

Thanks
 Adam Tyson
PowerScore Staff
  • PowerScore Staff
  • Posts: 5153
  • Joined: Apr 14, 2011
|
#49693
In this Weaken-X question, HowardQ, we need to pick the answer that does NOT weaken the argument. Anything that hurts the claim that "no one should ever follow any recommendations by these so called experts" will weaken, and thus be a wrong answer. So anything that makes us think that either the experts might be making good recommendations, or anything that makes us question the evidence used to support that conclusion, will be a loser.

Answer C attacks the data used in the premises. If the two things being compared - performance of the stocks recommended by the experts and performance of the market generally - were being measured using completely different standards, then the comparison isn't a fair one, and the data is no longer useful in supporting the conclusion. Maybe we should follow these experts' advice after all, for we might want dividends rather than increased share prices?

To use a simple analogy, this is like saying my friend recommended a restaurant to me, and I went and the service was slow, so I shouldn't listen to my friend about restaurants any more. If he had made the recommendation based on it being a romantic environment with a good wine selection, then my rejecting him on the grounds of slow service isn't supported. Maybe I should still listen to him if I am looking for romance and wine?

Attacking the data, or showing that the premises may not be relevant or applicable, is one way to weaken an argument. C does that nicely!
User avatar
 yycstudent
  • Posts: 9
  • Joined: Jul 20, 2022
|
#96283
Hello,

This question is a part of the Drill Test for "Weaken Questions: Volume 2" and the correct answer (as per the digital checkmark system) for the WeakenX is D "
Performance of the stocks recommended on the television show was measured independently by a number of analysts, and the results of all the measurements concurred."
However, C should be the correct answer as per the above discussion - OR am I misreading something?

Thank you.
 Rachael Wilkenfeld
PowerScore Staff
  • PowerScore Staff
  • Posts: 1358
  • Joined: Dec 15, 2011
|
#96308
You may be misreading things a bit, yycstudent. Answer choice (C) DOES weaken the argument in the stimulus. We aren't looking for a weaken answer choice here. We are looking for an answer choice that does not weaken because this is a weaken except question. Four incorrect answer choices will weaken, while our one correct answer choice will not weaken. Answer choice (D) on the other hand does not weaken the argument as it supports the data from the stimulus.

Hope that helps!

Get the most out of your LSAT Prep Plus subscription.

Analyze and track your performance with our Testing and Analytics Package.